LSAT and Law School Admissions Forum

Get expert LSAT preparation and law school admissions advice from PowerScore Test Preparation.

 cindyhylee87
  • Posts: 29
  • Joined: May 21, 2017
|
#42736
Hi Powerscore,

I am just wondering why the Justify Formula cannot be :

Reduce spending ..... :arrow: ..... Maintain quality

why it has to be he other way around?

It seems to me that
Reduce spending ..... :arrow: ..... Maintain quality
would justify the conclusion too.

Thanks,
Cindy




Administrator wrote:Complete Question Explanation

Justify the Conclusion—SN. The correct answer choice is (D)

This is a rather challenging question, in part because the premises are split between the first and the third sentences of the stimulus. The conclusion resides in the second sentence (note the conclusion indicator “so”), and the entire argument is conditional in nature. After applying the Unless Equation to the last sentence, the argument can be diagrammed as follows:
  • Premise (1): ..... Increase enrollment ..... :arrow: ..... Reduce spending

    Premise (2): ..... Increase enrollment ..... :arrow: ..... Marketing

    Conclusion: ..... Maintain quality ..... :arrow: ..... Marketing
The question stem asks us to identify a statement that, if assumed, would allow the conclusion to be properly drawn. Despite the word “assumed” in the stem, this is a Justify question because our job is not to identify a statement upon which the argument depends, but rather to prove the conclusion by adding a piece of information to the premises. The sufficient condition indicator (“if”) in the question stem is a reminder that you must select an answer that is sufficient to prove the conclusion by using the Justify Formula:
  • Premises + Answer choice = Conclusion
As with most Justify questions, there is a logical gap between the premises and the conclusion. To identify this gap easily, first connect the two premises using the contrapositive of the first:
  • Premise (1) + (2): ..... Reduce spending ..... :arrow: ..... Increase enrollment ..... :arrow: ..... Marketing
Now compare the additive inference to the conclusion. The premises suggest that to avoid having to reduce spending next year, the university needs to market its programs more aggressively. The conclusion, however, claims that aggressive marketing is necessary for maintaining the quality of education. To justify the conclusion, we need to establish a connection between maintaining the quality of education and not having to reduce spending next year:
  • Premise (1) + (2): ..... Reduce spending ..... :arrow: ..... Marketing

    Justify Formula: ..... Maintain quality ..... :arrow: ..... Reduce spending

    Conclusion: ..... Maintain quality ..... :arrow: ..... Marketing
In other words, we are looking for a statement suggesting that the quality of education will be maintained only if the university does not reduce spending next year. This prephrase is the contrapositive of answer choice (D), which is the correct answer choice.

The correct answer choice can also be arrived at by the process of elimination. First, given that the conclusion introduces a new, “rogue” element into the argument (“maintain quality of education”), the correct answer choice must connect that element to the rest of the argument. This eliminates answer choices (B), (C), and (E). Second, “increasing enrollment” is a term common to both of the premises but not to the conclusion of the argument. Therefore, the correct answer choice need not restate it, which eliminates answer choices (A), (B), and (C).

Answer choice (A): The contrapositive form of this answer choice suggests that the quality of education will be maintained only if the university does not increase its enrollment:
  • Maintain quality ..... :arrow: ..... Increase enrollment
This is markedly different from our prephrase, as it justifies a different conclusion. When combined with the premises, answer choice (A) only proves that maintaining the quality of education requires reducing spending next year:
  • Maintain quality ..... :arrow: ..... Increase enrollment ..... :arrow: ..... Reduce spending
Not only is this a different conclusion from the one we need to justify, but it is also the logical opposite of our Justify Formula.

It is also worth noting that “increasing enrollment” is a term common to both of the premises but not to the conclusion of the argument. Therefore, the correct answer choice need not restate it, making answer choice (A) relatively easy to eliminate.

Answer choice (B): This answer choice can be immediately eliminated because it does not specify a requirement necessary for the quality of education to be maintained. In other words, it does not connect the rogue element in the conclusion to the rest of the argument. It is also worth noting that this is the Mistaken Negation form of the first premise.
  • Answer choice (B): ..... Increase enrollment ..... :arrow: ..... Reduce spending

    Premise (1): ..... Increase enrollment ..... :arrow: ..... Reduce spending
Answer choice (C): Like incorrect answer choice (B), this answer choice can be eliminated because it does not connect “maintain quality of education” to the rest of the argument. Also, this is a Mistaken Reversal of the second premise.
  • Answer choice (C): ..... Marketing ..... :arrow: ..... Increase enrollment

    Premise (2): ..... Increase enrollment ..... :arrow: ..... Marketing
Answer choice (D): This is the correct answer choice, as it is identical to our prephrase:
  • Justify Formula: ..... Maintain quality ..... :arrow: ..... Reduce spending
    Contrapositive: ..... Reduce spending ..... :arrow: ..... Maintain quality
When combined with the premises, this statement forms a logical chain sufficient to prove the conclusion. Note that there are many ways to introduce this conditional relationship:
  • The university cannot both reduce spending and maintain the quality of education it provides.

    Unless the university avoids having to reduce spending next year, the quality of education it provides will not be maintained.

    The university will maintain the quality of education it provides only if it avoids having to reduce spending next year.
Answer choice (E): Like incorrect answer choices (B) and (C), this answer choice can be eliminated because it does not connect “maintain quality of education” to the rest of the argument. Also, it is the Mistaken Reversal form of the two premises, when combined:
  • Answer choice (E): ..... Marketing ..... :arrow: ..... Reduce spending

    Premise (1) + (2): ..... Reduce spending ..... :arrow: ..... Marketing
 Adam Tyson
PowerScore Staff
  • PowerScore Staff
  • Posts: 5153
  • Joined: Apr 14, 2011
|
#42796
Hey there, Cindy, let me see if I can help. The Justify formula works by providing a missing link in a chain, and that chain is often best expressed conditionally (think of them all as "if the premises are true, then the conclusion must be true"). Let's set up your hypothetical and see if it builds the right chain for us.

Our premises are increase enrollment :arrow: reduce spending and increase enrollment :arrow: marketing. These things don't link, as written, because none of the terms are the same. We can create a link, however, using a contrapositive of one or the other claim. I'll take the contrapositive of the first claim, which would be reduce spending :arrow: increase enrollment, and now I can connect it to the second claim and get this chain:

reduce spending :arrow: increase enrollment :arrow: marketing

So far, so good?

Now, the conclusion that we want to Justify is maintain quality :arrow: marketing

If I were to plug in your hypothetical solution of reduce spending :arrow: maintain quality (which is a reversal of what we have suggested you do), where would that go in my chain? Failing to reduce spending would be sufficient for two things - increasing enrollment and also maintaining quality. How does that connect quality to marketing? It doesn't - they are on two different "branches" of our conditional tree!

Now try it our way - if maintaining quality is sufficient for not reducing spending (maintain quality :arrow: reduce spending), then I can connect it all together in one continuous chain, like so:

maintain quality :arrow: reduce spending :arrow: increase enrollment :arrow: marketing

Now I have proven our conclusion - maintaining quality is indeed sufficient for marketing! Just follow the chain, connect the dots, and it all flows together!

Going backwards won't get you there, because the links need to all flow in one direction to get from point A to point D. Doing it the other way leads to two different branches that do not connect, and that is no way to justify anything.

Give that a try on a few more questions, and you will likely see a lot of wrong answers that connect the correct elements but in the wrong order, as you attempted to do. Those are attractive wrong answers, laid out there to trap you, so be careful! Make sure your chains are flowing all in the right direction, and you won't fall for those traps.

Good luck, and stay safe out there!
 ericau02
  • Posts: 73
  • Joined: Feb 19, 2019
|
#64714
Hi I am a little confused with this original diagram it seems s if it is off? it is really confusing me can anyone help or can it be re diagrammed ?
 ericau02
  • Posts: 73
  • Joined: Feb 19, 2019
|
#64715
I looked at another diagram below which helped me understand it much better, the only remaining question I have is when it come to conditional reasoning and questions like this is this a formula? for example am i alwasy to combine the premises and chain this with the contrapositive of the conclusion. And if so is the solely for justify the conclusions questions?
 Brook Miscoski
PowerScore Staff
  • PowerScore Staff
  • Posts: 418
  • Joined: Sep 13, 2018
|
#64987
erica, I think you tried to post a diagram but didn't include it so that it displays. Please repost your question with the diagram you are asking about.
 StephLewis13
  • Posts: 13
  • Joined: Jul 29, 2020
|
#78438
Hello,

I got this question right because I knew I had to link "Maintain Quality" as it was the "outsider element," but I really struggled because I set up the equation for Premise 2 incorrectly (as a mistaken reversal):

Market Aggressively —> Increase Enrollment

At first I was wrong because I thought 'maybe without is indicative of the sufficient or maybe there are exceptions.' But then I confirmed "without" was indicative of the necessary. After I read your explanation, it appears you must perform the "UNLESS Formula" when using "without." Is this correct? Is it also required for "Until" and "Except?" Are there any others that you must perform the UNLESS Formula on? Thank you!


Steph
 Jeremy Press
PowerScore Staff
  • PowerScore Staff
  • Posts: 1000
  • Joined: Jun 12, 2017
|
#78521
Hi Steph,

Good job correcting yourself here (that trait will serve you well as you continue to study)!

The terms you identified are indeed the ones on which the Unless Equation should be utilized: unless, except, until, without.

I never want to be too exhaustive in the way that I talk about formulas (the LSAT's writers have an incredible knack for finding new ways to say the same things), but off the top of my head I can't think of many (if any) instances of other terms on which I've had to use the Unless Equation on the test. One potential (quite stilted) phrase I can imagine them throwing in to test people at some point: "But for." Here's an example: "No one gets an F in this class but for laziness." That would diagram the same as: "No one gets an F in this class except the lazy." Both would get the following diagram:
Get an F :arrow: Lazy.

In general, though, you will be fine so long as you notice the four terms above.

I hope this helps!

Get the most out of your LSAT Prep Plus subscription.

Analyze and track your performance with our Testing and Analytics Package.